LSAT and Law School Admissions Forum

Get expert LSAT preparation and law school admissions advice from PowerScore Test Preparation.

 mcdonom4
  • Posts: 19
  • Joined: Mar 09, 2017
|
#34031
Hello!

I was just wondering what type of question this is? I'm torn between strengthen and mbt.
User avatar
 Dave Killoran
PowerScore Staff
  • PowerScore Staff
  • Posts: 5850
  • Joined: Mar 25, 2011
|
#34055
Hi McD,

Thanks for the question! You are right on target with your thinking, so let's break it down further and see what they are doing here.

The stimulus here features two speakers—Rossi and Smith—having a discussion about children's voting rights, and Rossi speaks first and then Smith replies. The question stem then reads:

  • "Smith’s statements can most directly be used as part of an argument for which one of the following views?"

That is unquestionably an unusual stem and not one you see very often at all, so it is understandable that it caused you to pause! Here, the test makers want to use Smith's statements as part of, or as the basis for, a second argument, and thus Smith's statement is equivalent to a premise of the new argument. In the case where one statement is a premise and the new argument will follow from that premise, the parallel situation from an informational standpoint is that the correct answer choice then results from the information in the stimulus. That's a classic First Family model (stimulus statements used to prove an answer choice), and so this is a Must Be True/Most Strongly Supported type of question. The Strengthen type you mention also makes some sense because the question stem wants you to use Smith's statement to strengthen a new argument. However, when that "strengthen" idea travels from the stimulus down to the answers, that actually fits the First Family model.

One interesting point here is that in a Second Family (Strengthen) question, the correct answer choice is used to strengthen or help the information in the stimulus. In a First Family (Must) question, one way to look at it is that the relationship is reversed (as indicated by the reversed direction of the arrow) and thus that the information in the stimulus is use to strengthen/prove the information in the correct answer.

Please let me know if that helps. Thanks!
 mcdonom4
  • Posts: 19
  • Joined: Mar 09, 2017
|
#34056
David,

Breaking it down like that totally makes sense that it's a Must Be True! Plus seeing which family a question belongs to is a good way to look at questions that are a little odd. Thank you so much! :)

Get the most out of your LSAT Prep Plus subscription.

Analyze and track your performance with our Testing and Analytics Package.